Supremum and Infimum

preview_player
Показать описание

Рекомендации по теме
Комментарии
Автор

Very very helpful explanation. After struggling 2 days finally I got the idea of the proof of general definition of supremum and infimum. Dear sir, Thank you from the button of my heart

RtGMUmEaRKING
Автор

One question on the definition of Supremum. Say there is a set S = [1, 4] and I take a = 3. So as per the definition given above. for all e>0, there exist (!) an x in S s.t. a-e < x <= a. So then 3 is a Supremum of [1, 4]. Which is definitely not the case. I guess we have to say for any upperbound a of set S, not just any element a...

somnathkundu
Автор

Fabulous
Very well elaborated made very easy to get about Infima and Suprema
Thanks a lot

VikasKumar-eetm
Автор

Nicely explained! Better than my teacher at IIT Kgp and understandable enough for a student on his first day!

SohamChakraborty
Автор

Nothing is interesting than proving theorems in real analysis 😁 I am in love with sk mapa though I am bsc general student.

lifegame
Автор

Excellent, I understand this clearly now, Im now familiar on the terms they used.

wevenlabesig
Автор

Thank you so much sir for such a clear explanation

vaishnavinair
Автор

Key concepts explained so well. Donnyavad!

punditgi
Автор

I have never seen real mathematice teaching like you

ramanujanclasses
Автор

Really good explanation, could I get the notes with the other proof for f(c)>0 but I don't know if it's actually correct. I followed your same steps but going from the right to the left. Thank you in advance

Albkiller
Автор

i have one doubt this topic, upper bound is also a lub, lower bound is take it as g l b

bhavyavenkataramua.v
Автор

very nice.send more and more in my mail

shyamalmanna
Автор

I like your lecture sir I want your notes so can you please give me any link to download it

sonukumar-
Автор

B + epsilon can't be lower bound
A - epsilon can't be upper bound
Max min must be elements of the set
Super inf may not be elements of the set
Prove intermediate value theorem using supremum infimum
Book spewak

alute
Автор

sir, i think their should be 1 - e < x < 1 + e. for x being the lub.

deepdhiman